LSAT and Law School Admissions Forum

Get expert LSAT preparation and law school admissions advice from PowerScore Test Preparation.

User avatar
 Dave Killoran
PowerScore Staff
  • PowerScore Staff
  • Posts: 5853
  • Joined: Mar 25, 2011
|
#80603
Complete Question Explanation
(The complete setup for this game can be found here: lsat/viewtopic.php?t=8622)

The correct answer choice is (D)

From the setup, we know that cities 1, 4, and 5 do not currently contain any of the institutions. The only remaining unplaced institutions are three hospitals and one jail (H, H, H, and J). But, we know that the jail requires a hospital, which effectively creates a JH block, leaving just three entities to cover the three empty cities:

G4-Q22-d1.png

Thus, although we cannot determine which city the jail is placed in, we can determine that there is a hospital placed in cities 1, 4, and 5. Thus, answer choice (D) must be true and is correct.
You do not have the required permissions to view the files attached to this post.
 josuecarolina
  • Posts: 24
  • Joined: Jul 20, 2012
|
#22257
Initially I had a very difficult time understanding the first rule, but got it cleared up from another post. Even after the clarification, this rule is poo. It makes it seem like there can't be a hospital, university or jail in more than one of the cities....But I get it.

Nonetheless, I did not get problem 23 and 24, and still do not understand them.

I don't even know where to start with my question, I got the other 4 right, and they made sense, but these last 2 are just slipping from my grasp.

Thanks in advance!
User avatar
 Dave Killoran
PowerScore Staff
  • PowerScore Staff
  • Posts: 5853
  • Joined: Mar 25, 2011
|
#22258
Hey Josue,

You've run into two tough questions here at the end of this section. Even though you answered question #22 correctly, I'm going to include an explanation of that question here because it plays into answering question #23 correctly.

Question #22: Local, Must Be True. The correct answer choice is (D)

From the setup, we know that cities 1, 4, and 5 do not currently contain any of the institutions. The only remaining unplaced institutions are three hospitals and one jail (H, H, H, and J). But, we know that the jail requires a hospital, which effectively creates a JH block, leaving just three entities to cover the three empty cities:


..... ..... ..... ..... ..... J H ..... H ..... H


Thus, although we cannot determine which city the jail is placed in, we can determine that there is a hospital placed in cities 1, 4, and 5. Thus, answer choice (D) must be true and is correct.

Please let me know if that helps. Thanks!

Get the most out of your LSAT Prep Plus subscription.

Analyze and track your performance with our Testing and Analytics Package.